Math, asked by swayamyadav94, 3 months ago

As shwon in figure, LK = 6 √2 then 1) MK = ? 2) ML = ? 3) MN

Attachments:

Answers

Answered by MaheswariS
12

\textbf{Given:}

\textsf{In the given figure,}

\mathsf{LK=6\sqrt{2}}

\textbf{To find:}

\mathsf{1.\;MK}

\mathsf{2.\;ML}

\mathsf{3.\;MN}

\textbf{Solution:}

\textsf{From the figure,}

\mathsf{In\;\triangle\,MLK}

\mathsf{cos30^\circ=\dfrac{LK}{MK}}

\mathsf{\dfrac{\sqrt3}{2}=\dfrac{6\sqrt2}{MK}}

\mathsf{\sqrt3\,MK=12\sqrt2}

\mathsf{MK=\dfrac{12\sqrt2}{\sqrt3}}

\mathsf{MK=4\sqrt3\sqrt2}

\implies\boxed{\mathsf{MK=4\sqrt6}}

\mathsf{In\;\triangle\,MLK}

\mathsf{sin30^\circ=\dfrac{ML}{MK}}

\mathsf{\dfrac{1}{2}=\dfrac{ML}{4\sqrt6}}

\mathsf{ML=4\sqrt6{\times}\dfrac{1}{2}}

\implies\boxed{\mathsf{ML=2\sqrt6}}

\mathsf{In\;\triangle\,MKN}

\mathsf{sin45^\circ=\dfrac{MK}{MN}}

\mathsf{\dfrac{1}{\sqrt2}=\dfrac{4\sqrt6}{MN}}

\implies\mathsf{MN=4\sqrt6\sqrt2}

\implies\mathsf{MN=4\sqrt{12}}

\implies\mathsf{MN=4(2\sqrt3)}

\implies\boxed{\mathsf{MN=8\sqrt3}}

Similar questions